Difference between revisions of "User:Nik"

From Ferienserie MMP2
Jump to: navigation, search
 
(57 intermediate revisions by 4 users not shown)
Line 1: Line 1:
== Foreword ==
+
Niklaus Messerli
I use \(Q\:/\:P\) for the transformed system instead of \(\widetilde{q}\:/\:\widetilde{p}\) because it's easier to write in Latex.  
+
nik@student.ethz.ch
== Problem ==
+
Let
+
\( \Phi \in C^\infty(\mathbb{R}^n) \)
+
have the property that the system
+
\( p_i = \frac{\partial}{\partial q_i} \Phi (q, Q) \)
+
has a unique smooth solution
+
\( Q = Q(q,p) \).  
+
  
Define
+
{| class="wikitable"
\( P_i(q,p) = - \frac{\partial}{\partial Q_i} \Phi (q, Q) | _{Q= Q(q,p)} \)
+
!Article
 
+
!Check
Let \( \{\cdot,\cdot\} \) be the Poisson bracket, such that
+
|-
\(  \{f,g\} = \sum_{j=1}^n \frac{\partial f}{\partial q_j} \frac{\partial g}{\partial p_j} - \frac{\partial f}{\partial p_j} \frac{\partial g}{\partial q_j} \)
+
| [[Aufgaben:Problem 1|Problem 1]]
 
+
| 1
Show that:
+
|-
 
+
| [[Aufgaben:Problem 2|Problem 2]]
I) \( \{Q_i(q,p), Q_j(q,p)\} = 0 \)
+
| a-c):1, d):0
 
+
|-
II) \( \{P_i (q,p), P_j(q,p)\} = 0 \)
+
| [[Aufgaben:Problem 3|Problem 3]]
 
+
| d) Wieso müssen wir zeigen dass "\(\{L_g: g \in G\}\) is a group under composition"? Wir wissen doch schon, dass \(L: G \rightarrow \mathrm{Sym}G\) und dass \(G\) und \(\mathrm{Sym}G\) Gruppen sind.
III) \( \{Q_i(q,p), P_j(q,p)\} = \delta_{ij} \)
+
|-
 
+
| [[Aufgaben:Problem 4|Problem 4]]
==Solution==
+
| 1
 
+
|-
===Important equations===
+
| [[Aufgaben:Problem 5|Problem 5]]
 
+
| 1
$$ \underbrace{\frac{\partial p_i}{\partial q_k}}_{0} = \frac{\partial}{\partial q_k}(\frac{\partial \Phi}{\partial q_i}(q, Q)) $$
+
|-
$$ = \sum_{s=1}^{n}(\frac{\partial^2 \Phi}{\partial q_i \partial q_s}(q, Q)\underbrace{\frac{\partial q_s}{\partial q_k}}_{\delta_{sk}} + \frac{\partial^2 \Phi}{\partial q_i \partial Q_s}(q, Q)\frac{\partial Q_s}{\partial q_k}) $$
+
| [[Aufgaben:Problem 6|Problem 6]]
$$ \Rightarrow 0 = \frac{\partial^2 \Phi}{\partial q_i \partial q_k}(q, Q) + \sum_{s=1}^{n}(\frac{\partial^2 \Phi}{\partial q_i \partial Q_s}(q, Q)\frac{\partial Q_s}{\partial q_k}) $$
+
| 1
$$ \tag{1} \Rightarrow \boxed{ -\frac{\partial^2 \Phi}{\partial q_i \partial q_k}(q, Q) = \sum_{s=1}^{n}(\frac{\partial^2 \Phi}{\partial q_i \partial Q_s}(q, Q)\frac{\partial Q_s}{\partial q_k}) } $$
+
|-
 
+
| [[Aufgaben:Problem 7|Problem 7]]
 
+
| 1
$$ \underbrace{\frac{\partial p_i}{\partial p_k}}_{\delta_{ik}} = \frac{\partial}{\partial p_k}(\frac{\partial \Phi}{\partial q_i}(q, Q)) $$
+
|-
$$ = \sum_{s=1}^{n}(\frac{\partial^2 \Phi}{\partial q_i \partial q_s}(q, Q)\underbrace{\frac{\partial q_s}{\partial p_k}}_{0} + \frac{\partial^2 \Phi}{\partial q_i \partial Q_s}(q, Q)\frac{\partial Q_s}{\partial p_k}) $$
+
| [[Aufgaben:Problem 8|Problem 8]]
$$ \tag{2} \boxed{ \Rightarrow \delta_{ik} = \sum_{s=1}^{n}(\frac{\partial^2 \Phi}{\partial q_i \partial Q_s}(q, Q)\frac{\partial Q_s}{\partial p_k}) } $$
+
| 2
 
+
|-
 
+
| [[Aufgaben:Problem 9|Problem 9]]
$$ \frac{\partial P_i}{\partial q_k} = \frac{\partial}{\partial q_k}(-\frac{\partial \Phi}{\partial Q_i}(q, Q)) $$
+
| 1
$$ = \sum_{s=1}^{n}(-\frac{\partial^2 \Phi}{\partial Q_i \partial q_s}(q, Q)\underbrace{\frac{\partial q_s}{\partial q_k}}_{\delta_{sk}} - \frac{\partial^2 \Phi}{\partial Q_i \partial Q_s}(q, Q)\frac{\partial Q_s}{\partial q_k}) $$
+
|-
$$ \tag{3} \boxed{ \Rightarrow \frac{\partial P_i}{\partial q_k} = -\frac{\partial^2 \Phi}{\partial Q_i \partial q_k}(q, Q) - \sum_{s=1}^{n}(\frac{\partial^2 \Phi}{\partial Q_i \partial Q_s}(q, Q)\frac{\partial Q_s}{\partial q_k}) } $$
+
| [[Aufgaben:Problem 10|Problem 10]]
 
+
| a by craven seems legit
 
+
|-
$$ \frac{\partial P_i}{\partial p_k} = \frac{\partial}{\partial p_k}(-\frac{\partial \Phi}{\partial Q_i}(q, Q)) $$
+
| [[Aufgaben:Problem 11|Problem 11]]
$$ = \sum_{s=1}^{n}(-\frac{\partial^2 \Phi}{\partial Q_i \partial q_s}(q, Q)\underbrace{\frac{\partial q_s}{\partial p_k}}_{0} - \frac{\partial^2 \Phi}{\partial Q_i \partial Q_s}(q, Q)\frac{\partial Q_s}{\partial p_k}) $$
+
| 1
$$ \tag{4} \Rightarrow \boxed{ \frac{\partial P_i}{\partial p_k} = -\sum_{s=1}^{n}(\frac{\partial^2 \Phi}{\partial Q_i \partial Q_s}(q, Q)\frac{\partial Q_s}{\partial p_k}) } $$
+
|-
 
+
| [[Aufgaben:Problem 12|Problem 12]]
 
+
| bitch please
===Solution I)===
+
|-
 
+
| [[Aufgaben:Problem 13|Problem 13]]
$$ \{Q_i, Q_j\} = \sum_{k=1}^{n}(\frac{\partial Q_i}{\partial q_k}\frac{\partial Q_j}{\partial p_k} - \frac{\partial Q_i}{\partial p_k}\frac{\partial Q_j}{\partial q_k}) $$
+
|
 
+
|-
 
+
| [[Aufgaben:Problem 14|Problem 14]]
===Solution II)===
+
|
 
+
|-
$$ \{P_i, P_j\} = \sum_{k=1}^{n}(\frac{\partial P_i}{\partial q_k}\frac{\partial P_j}{\partial p_k} - \frac{\partial P_i}{\partial p_k}\frac{\partial P_j}{\partial q_k}) $$
+
| [[Aufgaben:Problem 15|Problem 15]]
 
+
| a): 1
 
+
|-
===Solution III)===
+
|}
 
+
$$ \{Q_i, P_j\} = \sum_{k=1}^{n}(\frac{\partial Q_i}{\partial q_k}\frac{\partial P_j}{\partial p_k} - \frac{\partial Q_i}{\partial p_k}\frac{\partial P_j}{\partial q_k}) $$
+
$$ = \sum_{k=1}^{n}(\frac{\partial Q_i}{\partial q_k}\frac{\partial}{\partial p_k}(-\frac{\partial \Phi}{\partial Q_j}(q, Q)) - \frac{\partial Q_i}{\partial p_k}\frac{\partial}{\partial q_k}(-\frac{\partial \Phi}{\partial Q_j}(q, Q))) $$
+
$$ = \sum_{k=1}^{n}(\frac{\partial Q_i}{\partial q_k}\frac{\partial}{\partial p_k}(-\frac{\partial \Phi}{\partial Q_j}(q, Q)) - \frac{\partial Q_i}{\partial p_k}\frac{\partial}{\partial q_k}(-\frac{\partial \Phi}{\partial Q_j}(q, Q))) $$
+

Latest revision as of 14:13, 3 August 2015

Niklaus Messerli nik@student.ethz.ch

Article Check
Problem 1 1
Problem 2 a-c):1, d):0
Problem 3 d) Wieso müssen wir zeigen dass "\(\{L_g: g \in G\}\) is a group under composition"? Wir wissen doch schon, dass \(L: G \rightarrow \mathrm{Sym}G\) und dass \(G\) und \(\mathrm{Sym}G\) Gruppen sind.
Problem 4 1
Problem 5 1
Problem 6 1
Problem 7 1
Problem 8 2
Problem 9 1
Problem 10 a by craven seems legit
Problem 11 1
Problem 12 bitch please
Problem 13
Problem 14
Problem 15 a): 1